Monotone matrix norms












12














[Ciarlet 2.2-10]




  1. Let $mathscr{S}_n$ be the set of symmetric matrices and $mathscr{S}_n^+$ the subset of non-negative definite symmetric matrices. A matrix norm $|cdot|$ to be monotone if
    $$Ainmathscr{S}_n^+; wedge; B-Ainmathscr{S}_n^+ Rightarrow |A| leq |B|.$$
    Show that the norms $|cdot|_2$ and $|cdot|_F$ (Frobenus norm) are monotone.

  2. More generally, show that if a matrix norm $|cdot|$ is invariant under unitary transformations, that is, if $|A| = |AU| = |UA|$ for every unitary matrix $U$, then it is monotone.

  3. Let $|cdot|$ be a monotone norm and $mbox{cond}(cdot)$ the condition number function associated with it. Prove that
    $$A,Binmathscr{S}_n^* Rightarrow mbox{cond}(A+B) leq maxleft{mbox{cond}(A),; mbox{cond}(B)right}$$
    where $mathscr{S}_n^*$ denotes the subset of positive definite symmetric matrices.




I already have proved (1), and I proved that $lambda_i(A) leq lambda_i(B)$, $forall i=1,2,ldots,n$ and $forall A,B-Ainmathscr{S}_n^+$. But I have had problems in order to prove (2) and (3). For (2), i proved that
begin{eqnarray*}
|A| & = & |U^*AU| = |mbox{diag}(lambda_i(A))|,\[0.3cm]
|B| & = & |V^*BV| = |mbox{diag}(lambda_i(B))|.
end{eqnarray*}
but I don't know what I should do next. Please help me and thanks in advance.










share|cite|improve this question





























    12














    [Ciarlet 2.2-10]




    1. Let $mathscr{S}_n$ be the set of symmetric matrices and $mathscr{S}_n^+$ the subset of non-negative definite symmetric matrices. A matrix norm $|cdot|$ to be monotone if
      $$Ainmathscr{S}_n^+; wedge; B-Ainmathscr{S}_n^+ Rightarrow |A| leq |B|.$$
      Show that the norms $|cdot|_2$ and $|cdot|_F$ (Frobenus norm) are monotone.

    2. More generally, show that if a matrix norm $|cdot|$ is invariant under unitary transformations, that is, if $|A| = |AU| = |UA|$ for every unitary matrix $U$, then it is monotone.

    3. Let $|cdot|$ be a monotone norm and $mbox{cond}(cdot)$ the condition number function associated with it. Prove that
      $$A,Binmathscr{S}_n^* Rightarrow mbox{cond}(A+B) leq maxleft{mbox{cond}(A),; mbox{cond}(B)right}$$
      where $mathscr{S}_n^*$ denotes the subset of positive definite symmetric matrices.




    I already have proved (1), and I proved that $lambda_i(A) leq lambda_i(B)$, $forall i=1,2,ldots,n$ and $forall A,B-Ainmathscr{S}_n^+$. But I have had problems in order to prove (2) and (3). For (2), i proved that
    begin{eqnarray*}
    |A| & = & |U^*AU| = |mbox{diag}(lambda_i(A))|,\[0.3cm]
    |B| & = & |V^*BV| = |mbox{diag}(lambda_i(B))|.
    end{eqnarray*}
    but I don't know what I should do next. Please help me and thanks in advance.










    share|cite|improve this question



























      12












      12








      12


      3





      [Ciarlet 2.2-10]




      1. Let $mathscr{S}_n$ be the set of symmetric matrices and $mathscr{S}_n^+$ the subset of non-negative definite symmetric matrices. A matrix norm $|cdot|$ to be monotone if
        $$Ainmathscr{S}_n^+; wedge; B-Ainmathscr{S}_n^+ Rightarrow |A| leq |B|.$$
        Show that the norms $|cdot|_2$ and $|cdot|_F$ (Frobenus norm) are monotone.

      2. More generally, show that if a matrix norm $|cdot|$ is invariant under unitary transformations, that is, if $|A| = |AU| = |UA|$ for every unitary matrix $U$, then it is monotone.

      3. Let $|cdot|$ be a monotone norm and $mbox{cond}(cdot)$ the condition number function associated with it. Prove that
        $$A,Binmathscr{S}_n^* Rightarrow mbox{cond}(A+B) leq maxleft{mbox{cond}(A),; mbox{cond}(B)right}$$
        where $mathscr{S}_n^*$ denotes the subset of positive definite symmetric matrices.




      I already have proved (1), and I proved that $lambda_i(A) leq lambda_i(B)$, $forall i=1,2,ldots,n$ and $forall A,B-Ainmathscr{S}_n^+$. But I have had problems in order to prove (2) and (3). For (2), i proved that
      begin{eqnarray*}
      |A| & = & |U^*AU| = |mbox{diag}(lambda_i(A))|,\[0.3cm]
      |B| & = & |V^*BV| = |mbox{diag}(lambda_i(B))|.
      end{eqnarray*}
      but I don't know what I should do next. Please help me and thanks in advance.










      share|cite|improve this question















      [Ciarlet 2.2-10]




      1. Let $mathscr{S}_n$ be the set of symmetric matrices and $mathscr{S}_n^+$ the subset of non-negative definite symmetric matrices. A matrix norm $|cdot|$ to be monotone if
        $$Ainmathscr{S}_n^+; wedge; B-Ainmathscr{S}_n^+ Rightarrow |A| leq |B|.$$
        Show that the norms $|cdot|_2$ and $|cdot|_F$ (Frobenus norm) are monotone.

      2. More generally, show that if a matrix norm $|cdot|$ is invariant under unitary transformations, that is, if $|A| = |AU| = |UA|$ for every unitary matrix $U$, then it is monotone.

      3. Let $|cdot|$ be a monotone norm and $mbox{cond}(cdot)$ the condition number function associated with it. Prove that
        $$A,Binmathscr{S}_n^* Rightarrow mbox{cond}(A+B) leq maxleft{mbox{cond}(A),; mbox{cond}(B)right}$$
        where $mathscr{S}_n^*$ denotes the subset of positive definite symmetric matrices.




      I already have proved (1), and I proved that $lambda_i(A) leq lambda_i(B)$, $forall i=1,2,ldots,n$ and $forall A,B-Ainmathscr{S}_n^+$. But I have had problems in order to prove (2) and (3). For (2), i proved that
      begin{eqnarray*}
      |A| & = & |U^*AU| = |mbox{diag}(lambda_i(A))|,\[0.3cm]
      |B| & = & |V^*BV| = |mbox{diag}(lambda_i(B))|.
      end{eqnarray*}
      but I don't know what I should do next. Please help me and thanks in advance.







      linear-algebra norm






      share|cite|improve this question















      share|cite|improve this question













      share|cite|improve this question




      share|cite|improve this question








      edited Feb 26 '17 at 3:21









      Martin Argerami

      124k1176174




      124k1176174










      asked Apr 27 '13 at 17:56









      FASCH

      652926




      652926






















          1 Answer
          1






          active

          oldest

          votes


















          0














          For part 2:



          To simplify notation, define a function $g:mathbb R^ntomathbb R$ by $g(x_1,ldots,x_n)=|text{diag}(x_j)|$. Since $|cdot|$ is a unitarily invariant matrix norm, we have that




          • $g$ is a norm on $mathbb R^n$


          • $g(x_1,ldots,x_n)=g(|x_1|,ldots,|x_n|)$ (this comes from the unitary invariance)


          • $g(x_1,ldots,x_n)=g(x_{sigma(1)},ldots,x_{sigma(n)})$ for any permutation $sigma$.



          Such a $g$ is called a gauge function.



          Now, if $tin[0,1]$, then (writing $x=(x_1,ldots,x_n)$)
          begin{align}
          g(tx_1,x_2,ldots,x_n)&=gleft(frac{1+t}2,x+frac{1-t}2(-x_1,x_2,ldots,x_nright)\ \
          &leqfrac{1+t}2,g(x)+frac{1-t}2g(-x_1,x_2,ldots,x_n)\ \
          &=frac{1+t}2g(x)+frac{1-t}2g(x)=g(x).
          end{align}
          Applying the above inductively, we get
          $$
          g(t_1x_1,ldots,t_nx_n)leq g(x)
          $$
          whenever $t_1,ldots,t_nin [0,1]$.



          Since $0leqlambda_j(A)leqlambda_j(B)$ for all $j$, we have $lambda_j(A)=t_jlambda_j(B)$ for appropriate $t_1,ldots,t_nin[0,1]$.
          Then
          begin{align}
          |A|&=|text{diag}(lambda_j(A))|=|text{diag}(t_j,lambda_j(B))|\ \
          &leq |text{diag}(lambda_j(B))|=|B|
          end{align}



          For part 3, I know of the original proof by Marshall and Olkin (1973). Assume $text{cond}(A)leqtext{cond}(B)$. Let $A'=A/|A|$, $B'=B/|B|$, and $t=frac{|A|}{|A|+|B|}$.



          We have, in the new notation, that $$tag{0}|(A')^{-1}|leq|(B')^{-1}|.$$ And
          $$tag{1}
          |tA'+(1-t)B'|leq t|A'|+(1-t)|B'|=1.
          $$
          Also, as the inverse is convex on the set of positive-definite matrices,
          $$tag{2}
          (tA'+(1-t)B')^{-1}leq t(A')^{-1}+(1-t)(B')^{-1}.
          $$
          Thus, using $(2)$ and $(0)$,
          begin{align}tag{3}
          |(tA'+(1-t)B')^{-1}|&leq|t(A')^{-1}+(1-t)(B')^{-1}|\
          &leq t|(A')^{-1}|+(1-t)|(B')^{-1}|\ &leq|(B')^{-1}|
          end{align}
          Now, combining $(1)$, and $(3)$,
          begin{align}tag{4}
          |tA'+(1-t)B'|,|(tA'+(1-t)B')^{-1}|&leq |(tA'+(1-t)B')^{-1}|leq|(B')^{-1}|.
          end{align}
          If we now use the definitions of $A'$ and $B'$, we get
          $$
          tA'+(1-t)B'=frac1{|A|+|B|},left(A+Bright), (B')^{-1}=|B|,B^{-1}.
          $$
          We may thus rewrite $(4)$ as
          $$
          |A+B|,|(A+B)^{-1}|leq |B|,|B^{-1}|,
          $$
          as desired.






          share|cite|improve this answer























            Your Answer





            StackExchange.ifUsing("editor", function () {
            return StackExchange.using("mathjaxEditing", function () {
            StackExchange.MarkdownEditor.creationCallbacks.add(function (editor, postfix) {
            StackExchange.mathjaxEditing.prepareWmdForMathJax(editor, postfix, [["$", "$"], ["\\(","\\)"]]);
            });
            });
            }, "mathjax-editing");

            StackExchange.ready(function() {
            var channelOptions = {
            tags: "".split(" "),
            id: "69"
            };
            initTagRenderer("".split(" "), "".split(" "), channelOptions);

            StackExchange.using("externalEditor", function() {
            // Have to fire editor after snippets, if snippets enabled
            if (StackExchange.settings.snippets.snippetsEnabled) {
            StackExchange.using("snippets", function() {
            createEditor();
            });
            }
            else {
            createEditor();
            }
            });

            function createEditor() {
            StackExchange.prepareEditor({
            heartbeatType: 'answer',
            autoActivateHeartbeat: false,
            convertImagesToLinks: true,
            noModals: true,
            showLowRepImageUploadWarning: true,
            reputationToPostImages: 10,
            bindNavPrevention: true,
            postfix: "",
            imageUploader: {
            brandingHtml: "Powered by u003ca class="icon-imgur-white" href="https://imgur.com/"u003eu003c/au003e",
            contentPolicyHtml: "User contributions licensed under u003ca href="https://creativecommons.org/licenses/by-sa/3.0/"u003ecc by-sa 3.0 with attribution requiredu003c/au003e u003ca href="https://stackoverflow.com/legal/content-policy"u003e(content policy)u003c/au003e",
            allowUrls: true
            },
            noCode: true, onDemand: true,
            discardSelector: ".discard-answer"
            ,immediatelyShowMarkdownHelp:true
            });


            }
            });














            draft saved

            draft discarded


















            StackExchange.ready(
            function () {
            StackExchange.openid.initPostLogin('.new-post-login', 'https%3a%2f%2fmath.stackexchange.com%2fquestions%2f374500%2fmonotone-matrix-norms%23new-answer', 'question_page');
            }
            );

            Post as a guest















            Required, but never shown

























            1 Answer
            1






            active

            oldest

            votes








            1 Answer
            1






            active

            oldest

            votes









            active

            oldest

            votes






            active

            oldest

            votes









            0














            For part 2:



            To simplify notation, define a function $g:mathbb R^ntomathbb R$ by $g(x_1,ldots,x_n)=|text{diag}(x_j)|$. Since $|cdot|$ is a unitarily invariant matrix norm, we have that




            • $g$ is a norm on $mathbb R^n$


            • $g(x_1,ldots,x_n)=g(|x_1|,ldots,|x_n|)$ (this comes from the unitary invariance)


            • $g(x_1,ldots,x_n)=g(x_{sigma(1)},ldots,x_{sigma(n)})$ for any permutation $sigma$.



            Such a $g$ is called a gauge function.



            Now, if $tin[0,1]$, then (writing $x=(x_1,ldots,x_n)$)
            begin{align}
            g(tx_1,x_2,ldots,x_n)&=gleft(frac{1+t}2,x+frac{1-t}2(-x_1,x_2,ldots,x_nright)\ \
            &leqfrac{1+t}2,g(x)+frac{1-t}2g(-x_1,x_2,ldots,x_n)\ \
            &=frac{1+t}2g(x)+frac{1-t}2g(x)=g(x).
            end{align}
            Applying the above inductively, we get
            $$
            g(t_1x_1,ldots,t_nx_n)leq g(x)
            $$
            whenever $t_1,ldots,t_nin [0,1]$.



            Since $0leqlambda_j(A)leqlambda_j(B)$ for all $j$, we have $lambda_j(A)=t_jlambda_j(B)$ for appropriate $t_1,ldots,t_nin[0,1]$.
            Then
            begin{align}
            |A|&=|text{diag}(lambda_j(A))|=|text{diag}(t_j,lambda_j(B))|\ \
            &leq |text{diag}(lambda_j(B))|=|B|
            end{align}



            For part 3, I know of the original proof by Marshall and Olkin (1973). Assume $text{cond}(A)leqtext{cond}(B)$. Let $A'=A/|A|$, $B'=B/|B|$, and $t=frac{|A|}{|A|+|B|}$.



            We have, in the new notation, that $$tag{0}|(A')^{-1}|leq|(B')^{-1}|.$$ And
            $$tag{1}
            |tA'+(1-t)B'|leq t|A'|+(1-t)|B'|=1.
            $$
            Also, as the inverse is convex on the set of positive-definite matrices,
            $$tag{2}
            (tA'+(1-t)B')^{-1}leq t(A')^{-1}+(1-t)(B')^{-1}.
            $$
            Thus, using $(2)$ and $(0)$,
            begin{align}tag{3}
            |(tA'+(1-t)B')^{-1}|&leq|t(A')^{-1}+(1-t)(B')^{-1}|\
            &leq t|(A')^{-1}|+(1-t)|(B')^{-1}|\ &leq|(B')^{-1}|
            end{align}
            Now, combining $(1)$, and $(3)$,
            begin{align}tag{4}
            |tA'+(1-t)B'|,|(tA'+(1-t)B')^{-1}|&leq |(tA'+(1-t)B')^{-1}|leq|(B')^{-1}|.
            end{align}
            If we now use the definitions of $A'$ and $B'$, we get
            $$
            tA'+(1-t)B'=frac1{|A|+|B|},left(A+Bright), (B')^{-1}=|B|,B^{-1}.
            $$
            We may thus rewrite $(4)$ as
            $$
            |A+B|,|(A+B)^{-1}|leq |B|,|B^{-1}|,
            $$
            as desired.






            share|cite|improve this answer




























              0














              For part 2:



              To simplify notation, define a function $g:mathbb R^ntomathbb R$ by $g(x_1,ldots,x_n)=|text{diag}(x_j)|$. Since $|cdot|$ is a unitarily invariant matrix norm, we have that




              • $g$ is a norm on $mathbb R^n$


              • $g(x_1,ldots,x_n)=g(|x_1|,ldots,|x_n|)$ (this comes from the unitary invariance)


              • $g(x_1,ldots,x_n)=g(x_{sigma(1)},ldots,x_{sigma(n)})$ for any permutation $sigma$.



              Such a $g$ is called a gauge function.



              Now, if $tin[0,1]$, then (writing $x=(x_1,ldots,x_n)$)
              begin{align}
              g(tx_1,x_2,ldots,x_n)&=gleft(frac{1+t}2,x+frac{1-t}2(-x_1,x_2,ldots,x_nright)\ \
              &leqfrac{1+t}2,g(x)+frac{1-t}2g(-x_1,x_2,ldots,x_n)\ \
              &=frac{1+t}2g(x)+frac{1-t}2g(x)=g(x).
              end{align}
              Applying the above inductively, we get
              $$
              g(t_1x_1,ldots,t_nx_n)leq g(x)
              $$
              whenever $t_1,ldots,t_nin [0,1]$.



              Since $0leqlambda_j(A)leqlambda_j(B)$ for all $j$, we have $lambda_j(A)=t_jlambda_j(B)$ for appropriate $t_1,ldots,t_nin[0,1]$.
              Then
              begin{align}
              |A|&=|text{diag}(lambda_j(A))|=|text{diag}(t_j,lambda_j(B))|\ \
              &leq |text{diag}(lambda_j(B))|=|B|
              end{align}



              For part 3, I know of the original proof by Marshall and Olkin (1973). Assume $text{cond}(A)leqtext{cond}(B)$. Let $A'=A/|A|$, $B'=B/|B|$, and $t=frac{|A|}{|A|+|B|}$.



              We have, in the new notation, that $$tag{0}|(A')^{-1}|leq|(B')^{-1}|.$$ And
              $$tag{1}
              |tA'+(1-t)B'|leq t|A'|+(1-t)|B'|=1.
              $$
              Also, as the inverse is convex on the set of positive-definite matrices,
              $$tag{2}
              (tA'+(1-t)B')^{-1}leq t(A')^{-1}+(1-t)(B')^{-1}.
              $$
              Thus, using $(2)$ and $(0)$,
              begin{align}tag{3}
              |(tA'+(1-t)B')^{-1}|&leq|t(A')^{-1}+(1-t)(B')^{-1}|\
              &leq t|(A')^{-1}|+(1-t)|(B')^{-1}|\ &leq|(B')^{-1}|
              end{align}
              Now, combining $(1)$, and $(3)$,
              begin{align}tag{4}
              |tA'+(1-t)B'|,|(tA'+(1-t)B')^{-1}|&leq |(tA'+(1-t)B')^{-1}|leq|(B')^{-1}|.
              end{align}
              If we now use the definitions of $A'$ and $B'$, we get
              $$
              tA'+(1-t)B'=frac1{|A|+|B|},left(A+Bright), (B')^{-1}=|B|,B^{-1}.
              $$
              We may thus rewrite $(4)$ as
              $$
              |A+B|,|(A+B)^{-1}|leq |B|,|B^{-1}|,
              $$
              as desired.






              share|cite|improve this answer


























                0












                0








                0






                For part 2:



                To simplify notation, define a function $g:mathbb R^ntomathbb R$ by $g(x_1,ldots,x_n)=|text{diag}(x_j)|$. Since $|cdot|$ is a unitarily invariant matrix norm, we have that




                • $g$ is a norm on $mathbb R^n$


                • $g(x_1,ldots,x_n)=g(|x_1|,ldots,|x_n|)$ (this comes from the unitary invariance)


                • $g(x_1,ldots,x_n)=g(x_{sigma(1)},ldots,x_{sigma(n)})$ for any permutation $sigma$.



                Such a $g$ is called a gauge function.



                Now, if $tin[0,1]$, then (writing $x=(x_1,ldots,x_n)$)
                begin{align}
                g(tx_1,x_2,ldots,x_n)&=gleft(frac{1+t}2,x+frac{1-t}2(-x_1,x_2,ldots,x_nright)\ \
                &leqfrac{1+t}2,g(x)+frac{1-t}2g(-x_1,x_2,ldots,x_n)\ \
                &=frac{1+t}2g(x)+frac{1-t}2g(x)=g(x).
                end{align}
                Applying the above inductively, we get
                $$
                g(t_1x_1,ldots,t_nx_n)leq g(x)
                $$
                whenever $t_1,ldots,t_nin [0,1]$.



                Since $0leqlambda_j(A)leqlambda_j(B)$ for all $j$, we have $lambda_j(A)=t_jlambda_j(B)$ for appropriate $t_1,ldots,t_nin[0,1]$.
                Then
                begin{align}
                |A|&=|text{diag}(lambda_j(A))|=|text{diag}(t_j,lambda_j(B))|\ \
                &leq |text{diag}(lambda_j(B))|=|B|
                end{align}



                For part 3, I know of the original proof by Marshall and Olkin (1973). Assume $text{cond}(A)leqtext{cond}(B)$. Let $A'=A/|A|$, $B'=B/|B|$, and $t=frac{|A|}{|A|+|B|}$.



                We have, in the new notation, that $$tag{0}|(A')^{-1}|leq|(B')^{-1}|.$$ And
                $$tag{1}
                |tA'+(1-t)B'|leq t|A'|+(1-t)|B'|=1.
                $$
                Also, as the inverse is convex on the set of positive-definite matrices,
                $$tag{2}
                (tA'+(1-t)B')^{-1}leq t(A')^{-1}+(1-t)(B')^{-1}.
                $$
                Thus, using $(2)$ and $(0)$,
                begin{align}tag{3}
                |(tA'+(1-t)B')^{-1}|&leq|t(A')^{-1}+(1-t)(B')^{-1}|\
                &leq t|(A')^{-1}|+(1-t)|(B')^{-1}|\ &leq|(B')^{-1}|
                end{align}
                Now, combining $(1)$, and $(3)$,
                begin{align}tag{4}
                |tA'+(1-t)B'|,|(tA'+(1-t)B')^{-1}|&leq |(tA'+(1-t)B')^{-1}|leq|(B')^{-1}|.
                end{align}
                If we now use the definitions of $A'$ and $B'$, we get
                $$
                tA'+(1-t)B'=frac1{|A|+|B|},left(A+Bright), (B')^{-1}=|B|,B^{-1}.
                $$
                We may thus rewrite $(4)$ as
                $$
                |A+B|,|(A+B)^{-1}|leq |B|,|B^{-1}|,
                $$
                as desired.






                share|cite|improve this answer














                For part 2:



                To simplify notation, define a function $g:mathbb R^ntomathbb R$ by $g(x_1,ldots,x_n)=|text{diag}(x_j)|$. Since $|cdot|$ is a unitarily invariant matrix norm, we have that




                • $g$ is a norm on $mathbb R^n$


                • $g(x_1,ldots,x_n)=g(|x_1|,ldots,|x_n|)$ (this comes from the unitary invariance)


                • $g(x_1,ldots,x_n)=g(x_{sigma(1)},ldots,x_{sigma(n)})$ for any permutation $sigma$.



                Such a $g$ is called a gauge function.



                Now, if $tin[0,1]$, then (writing $x=(x_1,ldots,x_n)$)
                begin{align}
                g(tx_1,x_2,ldots,x_n)&=gleft(frac{1+t}2,x+frac{1-t}2(-x_1,x_2,ldots,x_nright)\ \
                &leqfrac{1+t}2,g(x)+frac{1-t}2g(-x_1,x_2,ldots,x_n)\ \
                &=frac{1+t}2g(x)+frac{1-t}2g(x)=g(x).
                end{align}
                Applying the above inductively, we get
                $$
                g(t_1x_1,ldots,t_nx_n)leq g(x)
                $$
                whenever $t_1,ldots,t_nin [0,1]$.



                Since $0leqlambda_j(A)leqlambda_j(B)$ for all $j$, we have $lambda_j(A)=t_jlambda_j(B)$ for appropriate $t_1,ldots,t_nin[0,1]$.
                Then
                begin{align}
                |A|&=|text{diag}(lambda_j(A))|=|text{diag}(t_j,lambda_j(B))|\ \
                &leq |text{diag}(lambda_j(B))|=|B|
                end{align}



                For part 3, I know of the original proof by Marshall and Olkin (1973). Assume $text{cond}(A)leqtext{cond}(B)$. Let $A'=A/|A|$, $B'=B/|B|$, and $t=frac{|A|}{|A|+|B|}$.



                We have, in the new notation, that $$tag{0}|(A')^{-1}|leq|(B')^{-1}|.$$ And
                $$tag{1}
                |tA'+(1-t)B'|leq t|A'|+(1-t)|B'|=1.
                $$
                Also, as the inverse is convex on the set of positive-definite matrices,
                $$tag{2}
                (tA'+(1-t)B')^{-1}leq t(A')^{-1}+(1-t)(B')^{-1}.
                $$
                Thus, using $(2)$ and $(0)$,
                begin{align}tag{3}
                |(tA'+(1-t)B')^{-1}|&leq|t(A')^{-1}+(1-t)(B')^{-1}|\
                &leq t|(A')^{-1}|+(1-t)|(B')^{-1}|\ &leq|(B')^{-1}|
                end{align}
                Now, combining $(1)$, and $(3)$,
                begin{align}tag{4}
                |tA'+(1-t)B'|,|(tA'+(1-t)B')^{-1}|&leq |(tA'+(1-t)B')^{-1}|leq|(B')^{-1}|.
                end{align}
                If we now use the definitions of $A'$ and $B'$, we get
                $$
                tA'+(1-t)B'=frac1{|A|+|B|},left(A+Bright), (B')^{-1}=|B|,B^{-1}.
                $$
                We may thus rewrite $(4)$ as
                $$
                |A+B|,|(A+B)^{-1}|leq |B|,|B^{-1}|,
                $$
                as desired.







                share|cite|improve this answer














                share|cite|improve this answer



                share|cite|improve this answer








                edited Apr 13 '17 at 12:21









                Community

                1




                1










                answered Feb 26 '17 at 3:21









                Martin Argerami

                124k1176174




                124k1176174






























                    draft saved

                    draft discarded




















































                    Thanks for contributing an answer to Mathematics Stack Exchange!


                    • Please be sure to answer the question. Provide details and share your research!

                    But avoid



                    • Asking for help, clarification, or responding to other answers.

                    • Making statements based on opinion; back them up with references or personal experience.


                    Use MathJax to format equations. MathJax reference.


                    To learn more, see our tips on writing great answers.





                    Some of your past answers have not been well-received, and you're in danger of being blocked from answering.


                    Please pay close attention to the following guidance:


                    • Please be sure to answer the question. Provide details and share your research!

                    But avoid



                    • Asking for help, clarification, or responding to other answers.

                    • Making statements based on opinion; back them up with references or personal experience.


                    To learn more, see our tips on writing great answers.




                    draft saved


                    draft discarded














                    StackExchange.ready(
                    function () {
                    StackExchange.openid.initPostLogin('.new-post-login', 'https%3a%2f%2fmath.stackexchange.com%2fquestions%2f374500%2fmonotone-matrix-norms%23new-answer', 'question_page');
                    }
                    );

                    Post as a guest















                    Required, but never shown





















































                    Required, but never shown














                    Required, but never shown












                    Required, but never shown







                    Required, but never shown

































                    Required, but never shown














                    Required, but never shown












                    Required, but never shown







                    Required, but never shown







                    Popular posts from this blog

                    Can a sorcerer learn a 5th-level spell early by creating spell slots using the Font of Magic feature?

                    Does disintegrating a polymorphed enemy still kill it after the 2018 errata?

                    A Topological Invariant for $pi_3(U(n))$